LESSON 2-1 PRACTICE

V 13. Attend to precision. Justify each step in the solution of 5x + 15 -

below by stating a property or providing an explanation for each

5x + 15 = 0

5x + 15 – 15 = 0 – 15

5x = -15

x = -3

Answers

Answer 1

Answer:

Value of x is -3

Step-by-step explanation:

Given:

5x + 15 = 0

Find:

Value of x

Computation:

5x + 15 = 0

From subtracting 15 from both side (LHS and RHS)

5x + 15 – 15 = 0 – 15

5x = -15

From dividing 5 from both side (LHS and RHS)

5x / 5 = -15 / 5

x = -3

So,

Value of x is -3


Related Questions

A shopkeeper allowed 10% discount on his goals
to make 20% profit If he sold a watch for
Rs.10,170 with 13 %. VAT, by what percent
is the discourt to be increased to make only
12% profit​

Answers

Answer:

To make only 12% profit, the discount should be increased to 16%

Step-by-step explanation:

The given information are;

The percentage discount on goods = 10%

The profit made from the sale of the watch = 20%

The amount the watch was sold = Rs.10,710

The percentage VAT included = 13%

Let the recommended selling price of the watch = X

Therefore, we have;

The price after 10% discount = 0.9×X

Which gives;

The amount the watch was sold = Price after 10% discount + Price after 10% discount ×VAT

The amount the watch was sold = 0.9×X + 0.9×X×13% = 0.9×X + 0.9×X×0.13

Rs.10,710 = 0.9×X + 0.9×X×0.13 = 1.017×X

X = Rs.10,170/1.017 = Rs. 10,000

The recommended selling price of the watch = Rs. 10,000

The profit was 20% of selling price which gives;

Profit = Selling price - Cost price = Rs. 9,000  - Rs. X = 0.2 × Rs. X

9000 - X = 0.2×X

9000 = X + 0.2×X  = 1.2×X

X = 9000/1.2 = Rs. 7,500

To make only 12% profit, we have;

12% of Rs. 7,500 = 0.12 × Rs.7,500 = Rs. 900

Selling price = Cost price + Profit = Rs. 7,500 + Rs. 900 = Rs. 8,400

Percentage discount from Rs.10,000 to get Rs. 8,400 is given as follows;

Percentage discount = Y%

Therefore;

Y = (10,000 - 8,400)/10000 × 100 = 0.16 × 100 = 16%

Y = 16%

Therefore, to make only 12% profit, the discount should be increased to 16%.

If the amount of VAT paid for an item at 13% was Rs 390, at what price was the item sold?​

Answers

Answer:

Step-by-step explanation:

Let the price of the item = Rs x

13% of x = 390

[tex]\frac{13}{100}*x=390\\\\\\x = 390*\frac{100}{13}\\\\\\[/tex]

x = Rs. 3000

Find m∠A.
A. 32
B. 26
C. 27
D. 30

Answers

Answer: B. 26

============================================

Use law of cosines to find angle A

a^2 = b^2 + c^2 - 2*b*c*cos(A)

9^2 = 6^2 + 14^2 - 2*6*14*cos(A)

81 = 36 + 196 - 168*cos(A)

81 = 232 - 168*cos(A)

81 - 232  = -168*cos(A)

-151  = -168*cos(A)

-168*cos(A) = -151

cos(A) = (-151)/(-168)

cos(A) = 0.8988095

A = arccos(0.8988095)

A = 25.9979801

A = 26 degrees approximately

The angle m∠A will be 27. so option C is correct.

What is a cosine law?

Cosine law is a formula relating the length of the sides of a triangle to the cosine of one angle of the triangle.

u² = s² + t² - 2(s)(t)·cos U

Use law of cosines to find angle A

a^2 = b^2 + c^2 - 2*b*c*cos(A)

9^2 = 6^2 + 14^2 - 2*6*14*cos(A)

81 = 36 + 196 - 168*cos(A)

81 = 232 - 168*cos(A)

81 - 232  = -168*cos(A)

-151  = -168*cos(A)

-168*cos(A) = -151

cos(A) = (-151)/(-168)

cos(A) = 0.8988095

A = 25.9979801

A = 26 degrees approximately

Therefore, the angle m∠A will be 27. so option C is correct.

Learn more about sine to cosine conversion here:

https://brainly.com/question/1421592

#SPJ5

Gary is using an indirect method to prove that segment DE is not parallel to segment BC in the triangle ABC shown below:

A triangle ABC is shown. D is a point on side AB and E is a point on side AC. Points D and E are joined using a straight line.

He starts with the assumption that segment DE is parallel to segment BC.

Which inequality will he use to contradict the assumption?

Answers

Answer:

That Ratios aren't equal.

Step-by-step explanation:

Given: DE is || to BC.

So in order to make this false then we have to say that the sides aren't proprtional, making it not possible to get the ratios equal.

(See the Triangle proprtionality theorem or the triangle midsegment theorem)      

Answer:

4:10 ≠ 6:14

Step-by-step explanation:

bc i said so

6) By which least number should each of the following numbers be divided to make them perfect squares? b) 45125

Answers

45125|5

9025|5

1805|5

361|19

19|19

1

[tex]45125=5^3\cdot 19^2=5^2\cdot 5\cdot19^2[/tex]

It's 5

whats the factored form of 6x 2 - 8x - 8 = 0

Answers

Answer:

x = -2/3 , 2

Step-by-step explanation:

Factor 2  out of   6 x^ 2 − 8 x − 8

2 ( 3 x^ 2 − 4 x − 4 ) = 0

Factor

2 ( 3 x + 2 ) ( x − 2 ) = 0

Set  3 x + 2  equal to  0  and solve for  x

x = -2/3

Set  x − 2  equal to  0  and solve for  x

x = 2

The final solution is all the values that make 2 ( 3 x + 2 ) ( x − 2 ) = 0

x = -2/3 , 2

Hope this can help you

 


A coin is tossed. What is the theoretical probability of the coin NOT showing tails?
P(Not tails) =

Answers

Answer:

50%

Step-by-step explanation:

its 50% it will land on head and 50% it will land on tails since there is only two sides on a coin

Answer:

1/2 or .5

p(1/2)

Step-by-step explanation:

its simple, there are 2 sides to a coin, so there are 2 possible outcomes. and the question asks what is the probability of the coin landing on one or in other wrds, its asking what is te probilitity of one of the two heads to be up. SO the probility is 1/2

PLZ ANSWER THIS QUESTION..Plzz

Answers

Answer:

x =  2 units

Step-by-step explanation:

Diagonals of rectangle are congruent and bisect each other

Therefore, OB = OA

   5x + 4 =  3x + 8

    5x = 3x + 8 -4

    5x = 3x + 4

5x - 3x = 4

      2x =  4

      x = 4/2

      x =2

Write an equation for a line on the graph that passes through the points (0.4) and (12,16)

Answers

Answer:

[tex] y = x + 4 [/tex]

Step-by-step explanation:

Use the two-point form of the equation of a line.

[tex] y - y_1 = \dfrac{y_2 - y_1}{x_2 - x_1}(x - x_1) [/tex]

[tex] y - 4 = \dfrac{16 - 4}{12 - 0}(x - 0) [/tex]

[tex] y - 4 = \dfrac{12}{12}x [/tex]

[tex] y - 4 = x [/tex]

[tex] y = x + 4 [/tex]

Answer:

y = x + 4

Step-by-step explanation:

An equation for a line looks like:

=> y = mx +b

=> In this equation "m" is the slope.

=> "b" is the y-intercept.

To find the slope:

=> y/x - y1/x1

=> 16/12 - 4/0

=> 16 -4 / 12 - 0

=> 12 / 12

=> 1

So, the slope is 1.

Now our equation looks like:

y = 1x + b

=> y = x + b

Let's take some the values of "x" and "y"  of (0,4)

So, our now look like:

=> 4 = 1 (0) + b

=> 4 = b

So, b (y-intercept) = 4

Now, our final equation is:

=> y = x + 4

The width of a rectangle measures (8.3c-8.4d)(8.3c−8.4d) centimeters, and its length measures (5.3c+4.8d)(5.3c+4.8d) centimeters. Which expression represents the perimeter, in centimeters, of the rectangle?

Answers

Answer:

P = 27.2c-7.2d

Step-by-step explanation:

It is given that,

The width of a rectangle is (8.3c-8.4d)

The length of a rectangle is (5.3c+4.8d)

The perimeter of a rectangle is equal to the sum of its all sides i.e.

P = 2(l+b)

P = 2(8.3c-8.4d+5.3c+4.8d)

P = 2[(8.3c+5.3c)+(4.8d-8.4d)]

P = 2(13.6c-3.6d)

P = 27.2c-7.2d

Hence, the expression that represents the perimeter of the rectangle is 27.2c-7.2d.

Mia’s house and her aunt’s house are 15.4 inches apart on the map. If every 4 inches on the map represents 10 miles, what is the actual distance from Mia’s house to her aunt’s house, to the nearest tenth of a mile? 2.6 6.2 38.5 61.6

Answers

Answer:

38.5 miles

Step-by-step explanation:

Proportions:

4 inches ⇔ 10 miles

15.4 inches ⇔ M miles

M = 15.4*10/4

M = 38.5 miles

Answer:  38.5    

Step-by-step explanation: cuz im smart


WikiTongues has
videos of 500 different
languages from around the
world. What percent of the
7,111 total languages
spoken globally
is represented on
WikiTongues?

Answers

Hey there! I'm happy to help!

We see that 500 languages is a certain percent of the 7,111 total languages. When working with percent equations, the word "is" means "equals", and "of" usually means "multiplied by" This means that 500 is equal to a certain percent multiplied by 7,111. We can use p to represent this certain percent and create an equation below.

500=7,111p

Let's flip the equation around so the p is on the left side. Variables are usually supposed to be on the left.

7,111p=500

We divide both sides by 7,111.

p=0.0703

As a percent, this is about 7.03%. Therefore, about 7.03% of the total languages spoken globally are represented on WikiTongues.

I hope that this helps! Have a wonderful day! :D

Answer:

7.03 %

Step-by-step explanation:

500 / 7,111 = 0.07031359865

0.07031359865  x 100 = 7.03 %

Find a linear inequality with the following solution set. Each grid line represents one unit.
Pllzzzzzzz help!!!!!!!!!!

Answers

The equation of the line is y = 3x - 4, so the inequality is y <= 3x - 4.

Answer:

y <= 3x - 4

Standard form: [tex]3x-y-4\geq 0[/tex]

Copy-paste: 3x-y-4>=0

What is the the product of (-1 - 3i) and it’s conjugate?

Answers

Answer:

10

Step-by-step explanation:

(-1 - 3i)(-1 + 3i) = 1 - 3i + 3i -9i²

1 - 9i²; i² = -1, therefore 1 - 9(-1) = 1 + 9 = 10

At dinner, 100 students pass through the cafeteria line and were served meals. 40 fish entrees and 60 pasta entrees were served to the students. A total of 20 students chose neither entree. Assuming all students were served zero, one, or two entrees, how many students were served two entrees

Answers

Answer:  20

Step-by-step explanation:

Given: Total students at the dinner = 100

Number of fish entrees = 40

Number of pasta entrees = 60

Number of students chose neither entree = 20

Now , Number of students chose either fish or pasta = (Total students) - (Number of students chose neither entree)

= 100-20

= 80

Now , Number of students chose either fish or pasta = (Number of fish entrees) + (Number of pasta entrees)- (Number of students chose both)

⇒ Number of students chose both = (Number of fish entrees) +(Number of pasta entrees)-(Number of students chose either fish or pasta)

= 40+60-80

= 20

Hence, the number of students were served two entrees = 20

20 students were served two entrees.

Given,

total student pass through cafeteria line and were served meal is 100.

No. of students choose fish entries is 40.

No. of students choose pasta entrees is 60.

No. of student choose neither entree is 20.

We have to calculate the no. of students served two entrees.

Now Number of students chose either fish or pasta will be,

[tex]N=100-20[/tex]

[tex]N=80[/tex]

Now no. of students choose both will be,

[tex]N=(fish\ entree+\ pasta \ entree )-Entree\ either \ pasta \ or \ fish[/tex]

[tex]N=60+40-80[/tex]

[tex]N=20[/tex]

Hence 20 students were served two entrees.

For more details follow the link:

https://brainly.com/question/24550771

is 1+isqrt3 a complex number

Answers

Answer:

Step-by-step explanation:

Yes, due to the presence of that operator " i "

Consider the circle of radius 10 centered at the origin. Find an equation of the line tangent to the circle at the point (6, 8)

Answers

Answer:

y = -3/4 x + 25/2

Step-by-step explanation:

x² + y² = 100

Take derivative with respect to x.

2x + 2y dy/dx = 0

2y dy/dx = -2x

dy/dx = -x/y

Evaluate at (6, 8).

dy/dx = -6/8

dy/dx = -3/4

Use point-slope form to write equation:

y − 8 = -3/4 (x − 6)

Simplify.

y − 8 = -3/4 x + 9/2

y = -3/4 x + 25/2

Convert from an improper fraction to a mixed number 17\6

Answers

━━━━━━━☆☆━━━━━━━

▹ Answer

2 5/6

▹ Step-by-Step Explanation

[tex]\frac{17}{6} \\\\6 * 2 = 12\\12 + 5 = 17\\\\2\frac{5}{6}[/tex]

Hope this helps!

CloutAnswers ❁

━━━━━━━☆☆━━━━━━━

Answer:

2 5/6

Step-by-step explanation:

17/6

6 goes into 17 2 times  

2*6 = 12

17-12 = 5  

There is a remainder of 5  this goes over the denominator

2 5/6

24 is 75% percent of what number?

Answers

Answer:

32

Step-by-step explanation:

We can call the number x. 75% can be expressed as 0.75 so we can write the following equation: 0.75x = 24. Dividing both sides by 0.75 to get rid of the coefficient of x, we get that x = 32.

Answer:

32.

Step-by-step explanation:

Let's say the number we are trying to find is x.

24 = 0.75x

0.75x = 24

x = 24 / 0.75

x = 32

Hope this helps!

FORM A CUBIC POLYNOMIAL WHOSE ZEROS ARE -3,-1 AND 2

Answers

[tex](x+3)(x+1)(x-2)=\\(x^2+x+3x+3)(x-2)=\\(x^2+4x+3)(x-2)=\\x^3-2x^2+4x^2-8x+3x-6=\\x^3+2x^2-5x-6[/tex]

Find the decimal number exactly halfway between 1.01 and 1.02.
please help

Answers

Answer:

1.015

Step-by-step explanation:

just find their average, which is adding them up and dividing by 2

(1.01 + 1.02)/2 = 1.015

The decimal number exactly halfway between 1.01 and 1.02 is 1.015.

What are decimals?

A decimal numeral system is the standard system for denoting integer and non-integer numbers. The way of denoting numbers in the decimal system is often referred to as decimal notation.

The given numbers are,

1.01 and 1.02

Now to find the halfway between the number we need to find their sum first and then the half of the sum.

So, the sum of number,

1.01 + 1.02 = 1.03

Now for half way divide the sum by 2

1.03/2 = 1.015

Hence, The decimal number exactly halfway between 1.01 and 1.02 is 1.015.

More about decimal :

https://brainly.com/question/825223

#SPJ2

Please help! offering 25 points, 5 stars, and a thanks. Ive asked this 3 times now

Answers

Answer:

17 quarters

Step-by-step explanation:

Let q = quarters

n = nickels

.25q + .05n = 5.90

we have 16 more nickels than quarters so add 16 quarters to make them equal

n = q+16

Substitute

.25q + .05( q+16) = 5.90

Distribute

.25q+.5q+.80=5.90

Combine like terms

.30q +.8 = 5.90

Subtract .8 from each side

.30q = 5.10

Divide each side by .3

.3q/.3 = 5.1/.3

q = 17

Answer:

Gisel have:

17

quarters

Step-by-step explanation:

1 nickel = 5 cents

1 quarter = 25 cents

1 dollar = 100 cents

5,90 dollars = 5,9*100 = 590 cents

then:

n = t + 16

5n + 25t = 590

n = quantity of nickels

t = quantity of quarters

5(t+16) + 25t = 590

5*t + 5*16 + 25t = 590

5t + 80 + 25t = 590

30 t = 590 - 80

30 t = 510

t = 510 / 30

t = 17

n = t + 16

n = 17 + 16

n = 33

Check:

5n + 25t = 590

5*33 + 25*17 = 590

165 + 425 = 590

Maria is buying new carpet for her bedroom .Her bedroom is in the shape of a square and the length of each side is 12 feet write and simplify an exponential express to find how much carpet she needs.

Answers

Answer:

well just do area, and since it's the same in each side 12×4= 144

In ΔABC,if a=6cm,b=5cm and c=4cm.find the value of ∠A.PLZ solve it fast plz help me out of it it's urgent

Answers

Answer:

the value of angle A is 82.819°

4:3=x:6, find the value of x please help me

Answers

Answer:

x=8

Step-by-step explanation:

4:3=x:6

Multiply the first set by 2

4*2 : 3*2

8:6

That means x =8

a plank is 2m long and30cm wide has volume of 0.018m. what is its thickness

Answers

I think the thickness of the plank is 0.03m

PLEASE HELP ME ASAP!! I NEED ANSWERS AND EXPLANATIONS FAST!!!

Answers

Answer:

d. [tex]\angle BIJ \cong \angle CGJ\ and\ \angle BJI \cong \angle CJG[/tex]

Step-by-step explanation:

Here, we are given a diagram in which there are two triangles to be considered:

[tex]\triangle CJG \ and\ \triangle BJI[/tex].

We can consider the whole diagram in the form as attached in the answer area.

To find:

The conditions to prove [tex]\triangle CJG \sim \triangle BJI[/tex].

Solution:

First of all, let us learn about AAA similarity.

AAA stands for Angle Angle Angle.

i.e. when we prove that the all the three corresponding angles of two triangles are equal ,the triangles are similar.

There is one more property of the triangles, that if two angles of two triangles are equal then the third angle of the triangles will also be equal to each other.

So, actually, to prove that [tex]\triangle CJG \sim \triangle BJI[/tex], we need the conditions that two corresponding angles of the triangles are equal to each other, then we can say that third angle will also be equal. Therefore the triangles will be similar to each other.

Let us consider the corresponding angles:

[tex]\angle BIJ \ and \ \angle CGJ\ ,\ \angle BJI \ and \ \angle CJG\ , \ \angle IBJ\ and \ \angle GCJ[/tex]

So, if we are given that any of the two above three pairs are equal/congruent to each other, we can prove [tex]\triangle CJG \sim \triangle BJI[/tex].

Therefore, the correct answer is:

d. [tex]\angle BIJ \cong \angle CGJ\ and\ \angle BJI \cong \angle CJG[/tex]

Erica can run 1 / 6 fraction of a kilometer in a minute. Her school is 3 / 4 of a kilometer away from her home. At this speed, how long would it take Erica to run home from school? answer quick plz

Answers

Answer:

the result is 4.5 minutes.

- Erica runs 1/6 km in a minute.

- The school is 3/1 km away from her home.

Step-by-step explanation:

f(x) = x^2. What is g(x)?

Answers

The point shown on g(x) is (1,3)

If x = 1 and y = 3, this means x is multiplied by 3 ( 1 x 3 = 3)

This makes g(x) 3x^2

The answer is A.

Please answer this question now

Answers

Answer:

m<C = 102°

Step-by-step explanation:

Step 1: find measure of arc BC.

According to the Inscribed angle theorem of a circle, an inscribed angle is half the measure of the arc it intercepts.

m<D intercepts arc ABC

thus, 80° = ½(120+BC)

Solve for BC. Multiply both sides by 2

80*2 = 120 + BC

160 = 120 + BC

BC = 160 - 120 = 40°

Step 2: Find m < A

According to the Inscribed angle theorem, m < A = ½ of arc BCD = ½(40 + 116)

m < A = 78°

Step 3: find m < C

m < A + m < C = 180 (opposite angles of an inscribed quadrilateral are supplementary)

m < C = 180 - 78 = 102°

Other Questions
f(x) = x2. What is g(x)? The length of a rectangle is represented by the function L(x) = 4x. The width of that same rectangle is represented by the function W(x) = 7x2 4x + 2. Which of the following shows the area of the rectangle in terms of x? (L + W)(x) = 7x2 + 2 (L + W)(x) = 7x2 8x + 2 (L W)(x) = 28x3 16x2 + 8x (L W)(x) = 28x3 4x + 2 !50 POINTS! GIVE REAL ANSWER OR I REPORT PLZ NO TROLLS! An executive drove from his home at an average speed of 35 mph to an airport where a helicopter was waiting. The executive then boarded the helicopter and flew to the corporate offices at an average speed of 78 mph. The entire distance from his home to the office was 169.5 miles. If the executive spent the same amount of time in the car as he did the helicopter, how long did it take the executive to get to work? NEED HELP URGENT!! Which of the following was NOT a major cause of the new Red Scare in America? A. The Korean War B. The U-2 Plane incident C. The Alger Hiss case D. The Rosenberg spy trial A student wants to create a 6.0V DC battery from a 1.5V DC battery. Can this be done using a transformer alone Firm J has net income of $77,605, sales of $935,000, and average total assets of $467,500. Required: Calculate Firm Js margin, turnover, and return on investment (ROI). System of linear inequalities An aphorism is a cleverly worded statement that is meant to trick the listener into believing a Lie True False You are an investor who wants to form a portfolio that lies to the right of the "optimal" minimum standard deviation portfolio on the efficient frontier. You must: 0 / 1 puntos Invest only in risky securities. Borrow money at the risk-free rate, invest in the minimum standard deviation portfolio and, in addition, only in risky securities. Borrow money at the risk-free rate and invest everything in the minimum standard deviation portfolio. Invest only in risk-free securities. The Bohr model pictures a hydrogen atom in its ground state as a proton and an electron separated by the distance a0 = 0.529 1010 m. The electric potential created by the electron at the position of the proton is The leadership role of women in social movements was likely an effect of:O A. Government programs that trained and supported women.O B. The organization of women voters.O C. Limited opportunities available to women in other areas of publiclife.O D. The growing number of women elected representatives. find the value of x? please help A piece of equipment (Asset class 15.0) was purchased by the Jones Construction Company. The cost basis was $300,000. Determine the ADS and GDS depreciation deduction for this property each year plz help me asap!!!!!What is the equation of the linear function represented by the table?xy5142111845y = negative x + 9y = negative x + 13y = x + 13y = x + 9 A small amount of solid calcium hydroxide is shaken vigorously in a test tube almost full of water until no further change occurs and most of the solid settles out. The resulting solution is:______. A rope, under a tension of 153 N and fixed at both ends, oscillates in a second-harmonic standing wave pattern. The displacement of the rope is given by . where at one end of the rope, is in meters, and is in seconds. What are (a) the length of the rope, (b) the speed of the waves on the rope, and (c) the mass of the rope? (d) If the rope oscillates in a third-harmonic standing wave pattern, what will be the period of oscillation? Dyesss book was published when the war was still being fought. Beck and Burgos were interviewed in the 1990s, five decades after the war. How did Dyesss account of the Bataan Death March differ from Becks and Burgoss accounts? How were the accounts similar? Describe your answer in at least three sentences. In the diagram, XY bisects ZWXZ.ZY2w(5x + 3)(7X - 70x=type your answer... URGENT PLEASE ANSWER !! A company has reported operating income of $25,000,000. The bond interest expense for the year is $4,000,000 and principal payments on bonds totaled $1,000,000. The company's debt service coverage ratio is: